Convolution is well defined in $L^1$












1












$begingroup$


I know that this question has been "answered" before, but I am struggling to understand the solutions previously given.



Let $f$ and $g$ be in $L^1(R, L, m)$



a) Show that $f*g$ is well defined for a.e. $x in R$. That is show $f(x-y)g(y)$ is integrable for all x.



b) Show that $ ||f*g||_1 le||f||_1||g||_1$



I have the following so far:



For part a)
$|int f(x-y)g(y)dy|le int |f(x-y)||g(y)|dy le ||f||_infty ||g||_1$ by Holder's Inequality?



$||f*g||_1 = intint f(x-y)g(y)dydx = intint f(x-y)g(y)dxdy$ by Fubini Tonelli



= $int g(y)int f(x-y)dxdy = int g(y)dy int f(x)dx = ||f||_1||g||_1 $ by translation invariance.



First in part b, why is it an inequality instead of an equality? Is this a typo?



I have seen arguments that the proof of b implies a too, or simply saying "apply Tonelli's theorem" to prove part a, but have yet to find anything concrete for an answer.










share|cite|improve this question









$endgroup$












  • $begingroup$
    I assume $L^1(R, L, m)$ is supposed to contain $L$-valued functions on $R$ that are measurable w.r.t. $m$. What can $L$ be for you? (i. e. only $mathbb{R}$ or $mathbb{C}$ or an arbitrary Banach space?)
    $endgroup$
    – 0x539
    Jan 7 at 19:57








  • 1




    $begingroup$
    For $b$, from the definition of $|.|_1$ you should have lots of $|.|$s. Adding these back will show you why its only an inequality, not an equality
    $endgroup$
    – Calvin Khor
    Jan 7 at 20:06
















1












$begingroup$


I know that this question has been "answered" before, but I am struggling to understand the solutions previously given.



Let $f$ and $g$ be in $L^1(R, L, m)$



a) Show that $f*g$ is well defined for a.e. $x in R$. That is show $f(x-y)g(y)$ is integrable for all x.



b) Show that $ ||f*g||_1 le||f||_1||g||_1$



I have the following so far:



For part a)
$|int f(x-y)g(y)dy|le int |f(x-y)||g(y)|dy le ||f||_infty ||g||_1$ by Holder's Inequality?



$||f*g||_1 = intint f(x-y)g(y)dydx = intint f(x-y)g(y)dxdy$ by Fubini Tonelli



= $int g(y)int f(x-y)dxdy = int g(y)dy int f(x)dx = ||f||_1||g||_1 $ by translation invariance.



First in part b, why is it an inequality instead of an equality? Is this a typo?



I have seen arguments that the proof of b implies a too, or simply saying "apply Tonelli's theorem" to prove part a, but have yet to find anything concrete for an answer.










share|cite|improve this question









$endgroup$












  • $begingroup$
    I assume $L^1(R, L, m)$ is supposed to contain $L$-valued functions on $R$ that are measurable w.r.t. $m$. What can $L$ be for you? (i. e. only $mathbb{R}$ or $mathbb{C}$ or an arbitrary Banach space?)
    $endgroup$
    – 0x539
    Jan 7 at 19:57








  • 1




    $begingroup$
    For $b$, from the definition of $|.|_1$ you should have lots of $|.|$s. Adding these back will show you why its only an inequality, not an equality
    $endgroup$
    – Calvin Khor
    Jan 7 at 20:06














1












1








1





$begingroup$


I know that this question has been "answered" before, but I am struggling to understand the solutions previously given.



Let $f$ and $g$ be in $L^1(R, L, m)$



a) Show that $f*g$ is well defined for a.e. $x in R$. That is show $f(x-y)g(y)$ is integrable for all x.



b) Show that $ ||f*g||_1 le||f||_1||g||_1$



I have the following so far:



For part a)
$|int f(x-y)g(y)dy|le int |f(x-y)||g(y)|dy le ||f||_infty ||g||_1$ by Holder's Inequality?



$||f*g||_1 = intint f(x-y)g(y)dydx = intint f(x-y)g(y)dxdy$ by Fubini Tonelli



= $int g(y)int f(x-y)dxdy = int g(y)dy int f(x)dx = ||f||_1||g||_1 $ by translation invariance.



First in part b, why is it an inequality instead of an equality? Is this a typo?



I have seen arguments that the proof of b implies a too, or simply saying "apply Tonelli's theorem" to prove part a, but have yet to find anything concrete for an answer.










share|cite|improve this question









$endgroup$




I know that this question has been "answered" before, but I am struggling to understand the solutions previously given.



Let $f$ and $g$ be in $L^1(R, L, m)$



a) Show that $f*g$ is well defined for a.e. $x in R$. That is show $f(x-y)g(y)$ is integrable for all x.



b) Show that $ ||f*g||_1 le||f||_1||g||_1$



I have the following so far:



For part a)
$|int f(x-y)g(y)dy|le int |f(x-y)||g(y)|dy le ||f||_infty ||g||_1$ by Holder's Inequality?



$||f*g||_1 = intint f(x-y)g(y)dydx = intint f(x-y)g(y)dxdy$ by Fubini Tonelli



= $int g(y)int f(x-y)dxdy = int g(y)dy int f(x)dx = ||f||_1||g||_1 $ by translation invariance.



First in part b, why is it an inequality instead of an equality? Is this a typo?



I have seen arguments that the proof of b implies a too, or simply saying "apply Tonelli's theorem" to prove part a, but have yet to find anything concrete for an answer.







measure-theory convolution






share|cite|improve this question













share|cite|improve this question











share|cite|improve this question




share|cite|improve this question










asked Jan 7 at 19:47









Math LadyMath Lady

1266




1266












  • $begingroup$
    I assume $L^1(R, L, m)$ is supposed to contain $L$-valued functions on $R$ that are measurable w.r.t. $m$. What can $L$ be for you? (i. e. only $mathbb{R}$ or $mathbb{C}$ or an arbitrary Banach space?)
    $endgroup$
    – 0x539
    Jan 7 at 19:57








  • 1




    $begingroup$
    For $b$, from the definition of $|.|_1$ you should have lots of $|.|$s. Adding these back will show you why its only an inequality, not an equality
    $endgroup$
    – Calvin Khor
    Jan 7 at 20:06


















  • $begingroup$
    I assume $L^1(R, L, m)$ is supposed to contain $L$-valued functions on $R$ that are measurable w.r.t. $m$. What can $L$ be for you? (i. e. only $mathbb{R}$ or $mathbb{C}$ or an arbitrary Banach space?)
    $endgroup$
    – 0x539
    Jan 7 at 19:57








  • 1




    $begingroup$
    For $b$, from the definition of $|.|_1$ you should have lots of $|.|$s. Adding these back will show you why its only an inequality, not an equality
    $endgroup$
    – Calvin Khor
    Jan 7 at 20:06
















$begingroup$
I assume $L^1(R, L, m)$ is supposed to contain $L$-valued functions on $R$ that are measurable w.r.t. $m$. What can $L$ be for you? (i. e. only $mathbb{R}$ or $mathbb{C}$ or an arbitrary Banach space?)
$endgroup$
– 0x539
Jan 7 at 19:57






$begingroup$
I assume $L^1(R, L, m)$ is supposed to contain $L$-valued functions on $R$ that are measurable w.r.t. $m$. What can $L$ be for you? (i. e. only $mathbb{R}$ or $mathbb{C}$ or an arbitrary Banach space?)
$endgroup$
– 0x539
Jan 7 at 19:57






1




1




$begingroup$
For $b$, from the definition of $|.|_1$ you should have lots of $|.|$s. Adding these back will show you why its only an inequality, not an equality
$endgroup$
– Calvin Khor
Jan 7 at 20:06




$begingroup$
For $b$, from the definition of $|.|_1$ you should have lots of $|.|$s. Adding these back will show you why its only an inequality, not an equality
$endgroup$
– Calvin Khor
Jan 7 at 20:06










2 Answers
2






active

oldest

votes


















2












$begingroup$

Actually, you do prove $b$ before proving $a$. Let $F(x,y)=f(x-y)g(y)$. $F$ is clearly measurable, and $|F|$ is clearly integrable over $mathbb{R}^2$, with integral $|f|_1|g|_1$ (Fubini-Tonelli theorem ensures that all manipulations are legit on the double integral of $|F|$ so you can integrate wrt $x$ first, make the change of variables $x=x’+y$).



Then you use Fubini’s theorem to prove that $x longmapsto int{F(x,y)dy}$ is well-defined and absolutely convergent ae and is in $L^1(mathbb{R})$ with norm at most $|f|_1|g|_1$.






share|cite|improve this answer









$endgroup$





















    1












    $begingroup$

    Suppose that $f, gin L^1$. Then
    $$int int |f(t)g(x-t)|, dt , dx = int |f(t)| int |g(x - t)|, dx
    = int f(t) |g|_1 , dt = |f|_1 |g|_1. $$






    share|cite|improve this answer











    $endgroup$














      Your Answer








      StackExchange.ready(function() {
      var channelOptions = {
      tags: "".split(" "),
      id: "69"
      };
      initTagRenderer("".split(" "), "".split(" "), channelOptions);

      StackExchange.using("externalEditor", function() {
      // Have to fire editor after snippets, if snippets enabled
      if (StackExchange.settings.snippets.snippetsEnabled) {
      StackExchange.using("snippets", function() {
      createEditor();
      });
      }
      else {
      createEditor();
      }
      });

      function createEditor() {
      StackExchange.prepareEditor({
      heartbeatType: 'answer',
      autoActivateHeartbeat: false,
      convertImagesToLinks: true,
      noModals: true,
      showLowRepImageUploadWarning: true,
      reputationToPostImages: 10,
      bindNavPrevention: true,
      postfix: "",
      imageUploader: {
      brandingHtml: "Powered by u003ca class="icon-imgur-white" href="https://imgur.com/"u003eu003c/au003e",
      contentPolicyHtml: "User contributions licensed under u003ca href="https://creativecommons.org/licenses/by-sa/3.0/"u003ecc by-sa 3.0 with attribution requiredu003c/au003e u003ca href="https://stackoverflow.com/legal/content-policy"u003e(content policy)u003c/au003e",
      allowUrls: true
      },
      noCode: true, onDemand: true,
      discardSelector: ".discard-answer"
      ,immediatelyShowMarkdownHelp:true
      });


      }
      });














      draft saved

      draft discarded


















      StackExchange.ready(
      function () {
      StackExchange.openid.initPostLogin('.new-post-login', 'https%3a%2f%2fmath.stackexchange.com%2fquestions%2f3065423%2fconvolution-is-well-defined-in-l1%23new-answer', 'question_page');
      }
      );

      Post as a guest















      Required, but never shown

























      2 Answers
      2






      active

      oldest

      votes








      2 Answers
      2






      active

      oldest

      votes









      active

      oldest

      votes






      active

      oldest

      votes









      2












      $begingroup$

      Actually, you do prove $b$ before proving $a$. Let $F(x,y)=f(x-y)g(y)$. $F$ is clearly measurable, and $|F|$ is clearly integrable over $mathbb{R}^2$, with integral $|f|_1|g|_1$ (Fubini-Tonelli theorem ensures that all manipulations are legit on the double integral of $|F|$ so you can integrate wrt $x$ first, make the change of variables $x=x’+y$).



      Then you use Fubini’s theorem to prove that $x longmapsto int{F(x,y)dy}$ is well-defined and absolutely convergent ae and is in $L^1(mathbb{R})$ with norm at most $|f|_1|g|_1$.






      share|cite|improve this answer









      $endgroup$


















        2












        $begingroup$

        Actually, you do prove $b$ before proving $a$. Let $F(x,y)=f(x-y)g(y)$. $F$ is clearly measurable, and $|F|$ is clearly integrable over $mathbb{R}^2$, with integral $|f|_1|g|_1$ (Fubini-Tonelli theorem ensures that all manipulations are legit on the double integral of $|F|$ so you can integrate wrt $x$ first, make the change of variables $x=x’+y$).



        Then you use Fubini’s theorem to prove that $x longmapsto int{F(x,y)dy}$ is well-defined and absolutely convergent ae and is in $L^1(mathbb{R})$ with norm at most $|f|_1|g|_1$.






        share|cite|improve this answer









        $endgroup$
















          2












          2








          2





          $begingroup$

          Actually, you do prove $b$ before proving $a$. Let $F(x,y)=f(x-y)g(y)$. $F$ is clearly measurable, and $|F|$ is clearly integrable over $mathbb{R}^2$, with integral $|f|_1|g|_1$ (Fubini-Tonelli theorem ensures that all manipulations are legit on the double integral of $|F|$ so you can integrate wrt $x$ first, make the change of variables $x=x’+y$).



          Then you use Fubini’s theorem to prove that $x longmapsto int{F(x,y)dy}$ is well-defined and absolutely convergent ae and is in $L^1(mathbb{R})$ with norm at most $|f|_1|g|_1$.






          share|cite|improve this answer









          $endgroup$



          Actually, you do prove $b$ before proving $a$. Let $F(x,y)=f(x-y)g(y)$. $F$ is clearly measurable, and $|F|$ is clearly integrable over $mathbb{R}^2$, with integral $|f|_1|g|_1$ (Fubini-Tonelli theorem ensures that all manipulations are legit on the double integral of $|F|$ so you can integrate wrt $x$ first, make the change of variables $x=x’+y$).



          Then you use Fubini’s theorem to prove that $x longmapsto int{F(x,y)dy}$ is well-defined and absolutely convergent ae and is in $L^1(mathbb{R})$ with norm at most $|f|_1|g|_1$.







          share|cite|improve this answer












          share|cite|improve this answer



          share|cite|improve this answer










          answered Jan 7 at 19:57









          MindlackMindlack

          4,910211




          4,910211























              1












              $begingroup$

              Suppose that $f, gin L^1$. Then
              $$int int |f(t)g(x-t)|, dt , dx = int |f(t)| int |g(x - t)|, dx
              = int f(t) |g|_1 , dt = |f|_1 |g|_1. $$






              share|cite|improve this answer











              $endgroup$


















                1












                $begingroup$

                Suppose that $f, gin L^1$. Then
                $$int int |f(t)g(x-t)|, dt , dx = int |f(t)| int |g(x - t)|, dx
                = int f(t) |g|_1 , dt = |f|_1 |g|_1. $$






                share|cite|improve this answer











                $endgroup$
















                  1












                  1








                  1





                  $begingroup$

                  Suppose that $f, gin L^1$. Then
                  $$int int |f(t)g(x-t)|, dt , dx = int |f(t)| int |g(x - t)|, dx
                  = int f(t) |g|_1 , dt = |f|_1 |g|_1. $$






                  share|cite|improve this answer











                  $endgroup$



                  Suppose that $f, gin L^1$. Then
                  $$int int |f(t)g(x-t)|, dt , dx = int |f(t)| int |g(x - t)|, dx
                  = int f(t) |g|_1 , dt = |f|_1 |g|_1. $$







                  share|cite|improve this answer














                  share|cite|improve this answer



                  share|cite|improve this answer








                  edited Jan 8 at 0:07

























                  answered Jan 7 at 20:01









                  ncmathsadistncmathsadist

                  43.2k261103




                  43.2k261103






























                      draft saved

                      draft discarded




















































                      Thanks for contributing an answer to Mathematics Stack Exchange!


                      • Please be sure to answer the question. Provide details and share your research!

                      But avoid



                      • Asking for help, clarification, or responding to other answers.

                      • Making statements based on opinion; back them up with references or personal experience.


                      Use MathJax to format equations. MathJax reference.


                      To learn more, see our tips on writing great answers.




                      draft saved


                      draft discarded














                      StackExchange.ready(
                      function () {
                      StackExchange.openid.initPostLogin('.new-post-login', 'https%3a%2f%2fmath.stackexchange.com%2fquestions%2f3065423%2fconvolution-is-well-defined-in-l1%23new-answer', 'question_page');
                      }
                      );

                      Post as a guest















                      Required, but never shown





















































                      Required, but never shown














                      Required, but never shown












                      Required, but never shown







                      Required, but never shown

































                      Required, but never shown














                      Required, but never shown












                      Required, but never shown







                      Required, but never shown







                      Popular posts from this blog

                      To store a contact into the json file from server.js file using a class in NodeJS

                      Redirect URL with Chrome Remote Debugging Android Devices

                      Dieringhausen